Exercícios Resolvidos de Método de Redução de Ordem

Ver Teoria

Enunciado

Considere a equação diferencial:

t 2 y ' ' - 5 t y ' + 9 y = 0   ,   t > 0

Encontre uma outra solução desta equação pelo método de redução de ordem que não seja y 1 vezes uma constante.

Passo 1

Esse tipo de problema é para quando queremos descobrir uma solução quando outra já foi dada. Nós vamos supor que a outra solução é a seguinte:

y 2 = v y 1

y 1 = t ³

y 2 = v t ³

Passo 2

Agora a gente precisa calcular as derivadas pra substituirmos na EDO:

y 2 = v t ³

y ' 2 = v ' t 3 + 3 v t ²

y ' ' 2 = v ' ' t 3 + 3 v ' t 2 + 3 v ' t 2 + 6 v t

y ' ' 2 = v ' ' t 3 + 6 v ' t 2 + 6 v t

Passo 3

Agora vamos substituir na equação geral:

t 2 y ' ' - 5 t y ' + 9 y = 0

t 2 ( v ' ' t 3 + 6 v ' t 2 + 6 v t ) - 5 t ( v ' t 3 + 3 v t 2 ) + 9 v t ³ = 0

Vamos simplificar esse carinha aí:

v ' ' t 5 + 6 v ' t 4 + 6 v t 3 - 5 v ' t 4 - 15 v t ³ + 9 v t ³ = 0

v ' ' t 5 + v ' t 4 = 0

v ' ' t + v ' = 0

Passo 4

Agora vamos usar o método propriamente dito, vamos reduzir a ordem fazendo:

v ' = w

v ' ' = w '

Vamos substituir na equação que encontramos no passo anterior:

w ' t + w = 0

Agora que encontramos uma EDO Separável e de 1ª ordem, podemos resolver:

d w d t t + w = 0

d w d t t = - w

1 w d w = - 1 t d t

Integrando as duas equações:

∫ 1 w d w = - ∫ 1 t d t

ln ⁡ w = ln ⁡ t - 1

w = 1 t

v ' = 1 t

v = l n ⁡ ( t )

v = l n ⁡ ( t ) é uma solução não constante. E assim temos a segunda solução da equação:

y 2 = t 3 l n ⁡ ( t )

Passo 5

Resposta

y 2 = t 3 l n ⁡ ( t ) é também é solução da equação.

Exercícios de Livros Relacionados

Encontre a solução geral da equação diferencial dada. y ' ' - 2 y ' + 10 y = 0
Se ar² + br + c = 0 tem raízes iguais r1, mostre queComo a última expressão à direita na Eq. (i) é nula quando r = r1, segue que exp(r1t) é uma solução de L[y] = ay′′ + by′ + cy = 0.
Use o método de redução de ordem para encontrar uma segunda solução da equação diferencial dada.
Use a fórmula de Abel [Eq. (23) da Seção 3.2] para mostrar que o wronskiano de duas soluções quaisquer da equação dada é em que c1 é constante.
Considere o problema de valor inicialEncontre as coordenadas do ponto de máximo (tM, yM) em função de b. Descreva a dependência em b de tM e de yM quando b aumenta.